Last visit was: 23 Apr 2024, 16:06 It is currently 23 Apr 2024, 16:06

Close
GMAT Club Daily Prep
Thank you for using the timer - this advanced tool can estimate your performance and suggest more practice questions. We have subscribed you to Daily Prep Questions via email.

Customized
for You

we will pick new questions that match your level based on your Timer History

Track
Your Progress

every week, we’ll send you an estimated GMAT score based on your performance

Practice
Pays

we will pick new questions that match your level based on your Timer History
Not interested in getting valuable practice questions and articles delivered to your email? No problem, unsubscribe here.
Close
Request Expert Reply
Confirm Cancel
SORT BY:
Date
Tags:
Show Tags
Hide Tags
User avatar
Senior Manager
Senior Manager
Joined: 26 Apr 2009
Affiliations: ACA, CPA
Posts: 311
Own Kudos [?]: 272 [39]
Given Kudos: 41
Location: Vagabond
Concentration: Finance, Treasury, Banking
Schools:BC
GMAT 2: 620
WE 1: Big4, Audit
WE 2: Banking
Send PM
User avatar
Senior Manager
Senior Manager
Joined: 01 Apr 2008
Posts: 392
Own Kudos [?]: 4074 [2]
Given Kudos: 18
Name: Ronak Amin
Schools: IIM Lucknow (IPMX) - Class of 2014
Send PM
User avatar
Senior Manager
Senior Manager
Joined: 26 Apr 2009
Affiliations: ACA, CPA
Posts: 311
Own Kudos [?]: 272 [1]
Given Kudos: 41
Location: Vagabond
Concentration: Finance, Treasury, Banking
Schools:BC
GMAT 2: 620
WE 1: Big4, Audit
WE 2: Banking
Send PM
User avatar
Intern
Intern
Joined: 30 Sep 2009
Posts: 20
Own Kudos [?]: 167 [1]
Given Kudos: 6
Send PM
Re: When the rate of inflation exceeds the rate of return on the most [#permalink]
1
Kudos
It should not be E....
I too got really confused and took some time to get it right between C and E.
A minor conclusion that can also be drawn from the stimuli is that, if somebody, invests in an instrument other than the most profitable investment, he is bound to get a higher negative return....
This excludes option E.
User avatar
Senior Manager
Senior Manager
Joined: 26 Apr 2009
Affiliations: ACA, CPA
Posts: 311
Own Kudos [?]: 272 [1]
Given Kudos: 41
Location: Vagabond
Concentration: Finance, Treasury, Banking
Schools:BC
GMAT 2: 620
WE 1: Big4, Audit
WE 2: Banking
Send PM
When the rate of inflation exceeds the rate of return on the most [#permalink]
1
Kudos
B and C are in a way similar but B is very vague. (becoming less profitable).
The argument is based on two aspects - Rate of inflation & most profitable investment.

When in doubt... i always try to rally around the key words given in the stem.
Hope this helps.
avatar
Intern
Intern
Joined: 16 Aug 2009
Posts: 3
Own Kudos [?]: 3 [2]
Given Kudos: 0
Send PM
Re: When the rate of inflation exceeds the rate of return on the most [#permalink]
2
Kudos
Its C.

The questions says that

P (loss) = Inflation percent - profit on most profitable investment

If a particular investment got a greater loss then it means that its not as profitable as most profitbale

To explain in numbers
investment made - 100
Inflation percent be 5%
Profit on most profitable be 3%

so the real value of investment should 105 but is only 103 and p is 2%

so if a particular investment has loss greater p that means that it is less profitable since the inflations stays the same for all investments......
User avatar
Intern
Intern
Joined: 25 Jul 2009
Posts: 8
Own Kudos [?]: 7 [0]
Given Kudos: 0
Send PM
Re: When the rate of inflation exceeds the rate of return on the most [#permalink]
This one is real toughie between B and C
B says the businesss is less proffitable which is correct. To make the investment go below the minimum percentage either the cost or the revenue has to decrease. Either way both will result in the profitibility of the business.

C is also kind of correct. Had it been 'the investment in question is less profitable than the most profitable investment available theortically' it would have made more sense because practically it might happen that all businesses have failed to at much deeper level than the business at hand.
So the business we are talking about perhaps becomes the most proffitable of all.
User avatar
Manager
Manager
Joined: 16 Jul 2009
Posts: 140
Own Kudos [?]: 2409 [0]
Given Kudos: 3
Send PM
Re: When the rate of inflation exceeds the rate of return on the most [#permalink]
Was stuck between B and C.. Thanks sniper your POE is really helpful to arrive at the right answer..
User avatar
Director
Director
Joined: 25 Aug 2007
Posts: 520
Own Kudos [?]: 5420 [0]
Given Kudos: 40
WE 1: 3.5 yrs IT
WE 2: 2.5 yrs Retail chain
Send PM
Re: When the rate of inflation exceeds the rate of return on the most [#permalink]
We can't bring new info to the argument. So, only A and C left.

Confused b/w A and C, as it is given that when the rate of inflation exceeds (not has exceeded - crucial point). Due to this I marked A.

It is my fault no to read the argument carefully that we are given two types of investments: most profitable and any.

Thanks snippertrader for the post.
avatar
Intern
Intern
Joined: 04 Feb 2010
Posts: 8
Own Kudos [?]: 1 [0]
Given Kudos: 4
Send PM
Re: When the rate of inflation exceeds the rate of return on the most [#permalink]
manojgmat wrote:
I chose B, why B cannot be answer?


There are TWO investments posited in the question. The first is "the most profitable investment". The second is "a particular investment". The second is also the investment under investigation.

B cannot be the answer because B assumes that the "particular investment" IS "the most profitable investment", but nothing in the question explicitly states that they are the same.
User avatar
Manager
Manager
Joined: 21 Jan 2010
Posts: 114
Own Kudos [?]: 326 [0]
Given Kudos: 38
Send PM
Re: When the rate of inflation exceeds the rate of return on the most [#permalink]
Xisiqomelir wrote:
manojgmat wrote:
I chose B, why B cannot be answer?


There are TWO investments posited in the question. The first is "the most profitable investment". The second is "a particular investment". The second is also the investment under investigation.

B cannot be the answer because B assumes that the "particular investment" IS "the most profitable investment", but nothing in the question explicitly states that they are the same.



Also, B cannot be right because a particular investment can become more profitable but still decline more than "that percentage", which is the difference between inflation rate the the ROI of the MOST profitable investment.
User avatar
Senior Manager
Senior Manager
Joined: 21 Dec 2010
Posts: 267
Own Kudos [?]: 1331 [0]
Given Kudos: 51
Send PM
Re: When the rate of inflation exceeds the rate of return on the most [#permalink]
'B cannot be the answer because B assumes that the "particular investment" IS "the most profitable investment", but nothing in the question explicitly states that they are the same'

good explanation this , i came to C in 1:59 min .
User avatar
Senior Manager
Senior Manager
Joined: 21 Dec 2010
Posts: 267
Own Kudos [?]: 1331 [1]
Given Kudos: 51
Send PM
Re: When the rate of inflation exceeds the rate of return on the most [#permalink]
1
Kudos
@ calvinhobbes , i don't understand how this is possible

''Also, B cannot be right because a particular investment can become more profitable but still decline more than "that percentage", which is the difference between inflation rate and the ROI of the MOST profitable investment''
User avatar
Manager
Manager
Joined: 16 Mar 2011
Posts: 137
Own Kudos [?]: 19 [1]
Given Kudos: 1
Send PM
Re: When the rate of inflation exceeds the rate of return on the most [#permalink]
1
Kudos
C is the answer because:
the stem says "When the rate of inflation exceeds the rate of return on the most profitable investment available, the difference between those two rates will be the percentage by which, at a minimum, the value of any investment will decline. If in such a circumstance the value of a particular investment declines by more than that percentage, it must be true that:

the most important phrase in the stem is "most profitable", the base of the %term in question is the comparison b/w inflation and the most profitable. so anything that is more than this % have to be compared to the "most profitable".

C is the only choice providing the specific base of comparison "most profitable",
User avatar
Director
Director
Joined: 08 May 2009
Status:There is always something new !!
Affiliations: PMI,QAI Global,eXampleCG
Posts: 552
Own Kudos [?]: 588 [0]
Given Kudos: 10
Send PM
Re: When the rate of inflation exceeds the rate of return on the most [#permalink]
Most profitable investment will have a profitability decline %.
if the profitability decline % is more for the investment in question,its obvious that the profitability of the investment will be lesser than the most profitable investment.

Hence C comes out clean in this.
User avatar
Manager
Manager
Joined: 14 Apr 2011
Posts: 144
Own Kudos [?]: 38 [0]
Given Kudos: 19
Send PM
Re: When the rate of inflation exceeds the rate of return on the most [#permalink]
was stuck between B and C. felt inclined to choose C but picked B as it seemed correct.

ruled out E as it seems out of scope/unclear as the "change": whatever that is - it is not mentioned.
VP
VP
Joined: 18 Dec 2017
Posts: 1170
Own Kudos [?]: 991 [0]
Given Kudos: 421
Location: United States (KS)
GMAT 1: 600 Q46 V27
Send PM
Re: When the rate of inflation exceeds the rate of return on the most [#permalink]
snipertrader wrote:
When the rate of inflation exceeds the rate of return on the most profitable investment available, the difference between those two rates will be the percentage by which, at a minimum, the value of any investment will decline. If in such a circumstance the value of a particular investment declines by more than that percentage, it must be true that _________

Which one of the following logically completes the argument?

A) the rate of inflation has risen

B) the investment in question is becoming less profitable

C) the investment in question is less profitable than the most profitable investment available

D) the rate of return on the most profitable investment available has declined

E) there has been a change in which particular investment happens to be the most profitable available

Source : LSAT


Let's say inflation = 10%
Return on Most Profitable investment = 8%

Difference = 2%

Any investment will decline by 2% at the minimum.

Let's the investment in question drops by 4% (greater than the the percentage) then how can we claim that this investment is less profitable than 8% investment?
We don't know the actual return possible of the investment which dropped by 4%.

Thoughts?
CEO
CEO
Joined: 07 Mar 2019
Posts: 2552
Own Kudos [?]: 1812 [0]
Given Kudos: 763
Location: India
WE:Sales (Energy and Utilities)
Send PM
Re: When the rate of inflation exceeds the rate of return on the most [#permalink]
snipertrader wrote:
When the rate of inflation exceeds the rate of return on the most profitable investment available, the difference between those two rates will be the percentage by which, at a minimum, the value of any investment will decline. If in such a circumstance the value of a particular investment declines by more than that percentage, it must be true that _________

Which one of the following logically completes the argument?

A) the rate of inflation has risen

B) the investment in question is becoming less profitable

C) the investment in question is less profitable than the most profitable investment available

D) the rate of return on the most profitable investment available has declined

E) there has been a change in which particular investment happens to be the most profitable available

Source : LSAT

The percentage in question is difference between inflation and the percentage return the most profitable investment makes. Based on this A and D are out. E is not necessarily true. Among B and C, latter must be true in that it gives a relative comparison of returns of the two investments - particular investment and the most profitable one.
Manager
Manager
Joined: 29 Apr 2022
Posts: 203
Own Kudos [?]: 36 [0]
Given Kudos: 277
Location: India
Concentration: Finance, Marketing
GMAT 1: 690 Q48 V35 (Online)
WE:Engineering (Manufacturing)
Send PM
Re: When the rate of inflation exceeds the rate of return on the most [#permalink]
Unable to understand.
Please help
User avatar
Non-Human User
Joined: 01 Oct 2013
Posts: 17206
Own Kudos [?]: 848 [0]
Given Kudos: 0
Send PM
Re: When the rate of inflation exceeds the rate of return on the most [#permalink]
Hello from the GMAT Club VerbalBot!

Thanks to another GMAT Club member, I have just discovered this valuable topic, yet it had no discussion for over a year. I am now bumping it up - doing my job. I think you may find it valuable (esp those replies with Kudos).

Want to see all other topics I dig out? Follow me (click follow button on profile). You will receive a summary of all topics I bump in your profile area as well as via email.
GMAT Club Bot
Re: When the rate of inflation exceeds the rate of return on the most [#permalink]
Moderators:
GMAT Club Verbal Expert
6917 posts
GMAT Club Verbal Expert
238 posts
CR Forum Moderator
832 posts

Powered by phpBB © phpBB Group | Emoji artwork provided by EmojiOne